LSAT 46 – Section 3 – Question 09

You need a full course to see this video. Enroll now and get started in less than a minute.

Target time: 1:32

This is question data from the 7Sage LSAT Scorer. You can score your LSATs, track your results, and analyze your performance with pretty charts and vital statistics - all with a Free Account ← sign up in less than 10 seconds

Question
QuickView
Type Tags Answer
Choices
Curve Question
Difficulty
Psg/Game/S
Difficulty
Explanation
PT46 S3 Q09
+LR
Most strongly supported +MSS
A
78%
165
B
3%
156
C
11%
161
D
4%
157
E
4%
155
136
148
160
+Medium 145.195 +SubsectionEasier

We know this is a most strongly supported question because of the question stem: The statements in the editorial, if true, most strongly support which one of the following?

Our stimulus starts by explaining that when folks in government find out that a public service isn’t being provided adequately, the most common response is to increase funding for that service. Makes sense! If the trash isn’t getting hauled away fast enough–buy more trash trucks!

We are then told that the least efficiently run bureaucracies most commonly receive an increase in funds. So there’s a leap that’s being made to connect these two statements. We know that when services are poor, funding for services increases. The editorial states that “because of this” (referential for the increase in funding for inadequate services) the least efficient bureaucracies most commonly receive boosts in funding. If we trace the logic here, we can see that the implication is that these poorly run bureaucracies are most likely responsible for inadequately provided services. This is the only conceivable way that the editorial could say that funding for these bureaucracies commonly increases and give the first sentence as a reason why this funding commonly increases (as indicated by the phrase “because of this”).

That’s our synthesis right there. When we look at these two sentences together, the clear implication is that inefficient bureaucracies are responsible for inadequate services.

Now let’s look at the answer choices:

Correct Answer Choice (A) What do you know! First answer choice out the gates matches up with our synthesis. This comprehensively lays out the implication of these two sentences when taken together.

Answer Choice (B) We don’t have any information about when or why legislators might reduce funding–so we have no information to support this answer choice.

Answer Choice (C) This is introducing the idea that legislators “repeatedly” boost funding during periods of inefficiency. We don’t have any information to support this. We just know that inefficient bureaucracies are the bureaucracies that most commonly receive boosts in funding, but we have no idea how these boosts occur (i.e. repeatedly or in lump sums).

Answer Choice (D) We have no information that indicates how services or bureaucracies fare as a result of funding boosts–so we have no information that supports this answer choice.

Answer Choice (E) We know nothing about the amount of money included in any of these funding boosts. We know there’s a correlation between being inefficient and receiving money, but we don’t know if there’s a correlation between the magnitude of inefficiency and the amount of money.

Take PrepTest

Review Results

Leave a Reply